GMAT1

This topic has expert replies
User avatar
Master | Next Rank: 500 Posts
Posts: 218
Joined: Wed Dec 11, 2013 4:02 am
Thanked: 3 times
Followed by:4 members

GMAT1

by [email protected] » Sun Mar 30, 2014 9:59 am

User avatar
Legendary Member
Posts: 1556
Joined: Tue Aug 14, 2012 11:18 pm
Thanked: 448 times
Followed by:34 members
GMAT Score:650

by theCodeToGMAT » Sun Mar 30, 2014 11:43 pm
Is it [spoiler]{A}[/spoiler]?
R A H U L

User avatar
Master | Next Rank: 500 Posts
Posts: 218
Joined: Wed Dec 11, 2013 4:02 am
Thanked: 3 times
Followed by:4 members

by [email protected] » Mon Mar 31, 2014 5:24 am
Yes!
Explanation please?

Regards,
mukherjee

Junior | Next Rank: 30 Posts
Posts: 23
Joined: Sat Jul 27, 2013 9:32 pm
Thanked: 2 times

by viny » Mon Mar 31, 2014 1:10 pm
Here is the explanation
The background is people with rds equipped radio can listen to those programs
Premise : stations increased but no of radios equipped with rds tech dnt increase
Conclusion : no of people hearing the rds program did not increase SIGNIFICANTLY
This is lying on the assumption that there are some ppl who had rds equipped radios before itself because stations grew and radios are constant

In Find assumption questions always choose an answer which if not true will make the conclusion impossible to be derived from the premise
When you read A it says there are people having rds equipped radios already but dnt Lsn to the rds program because of no reach so this is a case with constant number of radios Also it says few people are like tht so even if u feel If those ppl who are living in those areas now after 1994 got access increased the total number verdlanders by a very small number .... Remember the argument doesn't say doesn't increase at all it says doesnt increase significantly ..
In love with "beatthegmat",I do not need any coaching I get all my doubt solved here on this 1 place...:) thanks to all the experts!!